SNAP Logarithms Questions PDF [Most Important]

0
909
_ Logarithms Questions
_ Logarithms Questions

SNAP Logarithms Questions PDF

Logarithms is an important topic in the Quant section of the SNAP Exam. You can also download this Free Logarithms Questions for SNAP PDF (with answers) by Cracku. These questions will help you to practice and solve the Logarithms questions in the SNAP exam. Utilize this PDF practice set, which is one of the best sources for practising.

Download Logarithms Questions for SNAP

Enroll to SNAP 2022 Crash Course

Question 1: Find the value of following expression $\log\sin 40^\circ \log \sin 41^\circ — \log \sin 99^\circ \log \sin 100^\circ$

a) $\frac{\sqrt{3ac}+1}{2}$

b) $0$

c) $1$

d) $2$

1) Answer (B)

Solution:

= $\log\sin 40^\circ \log \sin 41^\circ….. \log \sin 90^\circ…… \log \sin 99^\circ \log \sin 100^\circ$

= $\log \sin 90^\circ$

= log 1

= 0

Answer is option B.

Question 2: The value of $\log_2 x$ which satisfy $6 – 9\log_{8}\left(\frac{4}{x}\right)^{\frac{1}{3}} – 8(\log_{256}x)^{\frac{2}{3}} – (\log_2 x^8)^{\frac{1}{3}} = 0$ is

a) 2

b) $\sqrt{2}$

c) 4

d) 8

2) Answer (D)

Solution:

$6-9\log_8\left(\frac{4}{x}\right)^{\frac{1}{3}}-8\left(\log_{256}x\right)^{\frac{2}{3}}-\left(\log_{2\ }x^8\right)^{\frac{1}{3}}=0$

$6-\log_24+\log_2\ x-2\left(\log_2x\right)^{\frac{2}{3}}-2\left(\log_2x\right)^{\frac{1}{3}}=0$

Let $\left(\log_2x\right)^{\frac{1}{3}}$ be t.

$4+t^3-2t^2-2t=0$

or, $\left(t-2\right)\left(t^2-2\right)=0$

so, $t=2$ or

$\log_2\ x=8$

Question 3: For a real number a, if $\frac{\log_{15}{a}+\log_{32}{a}}{(\log_{15}{a})(\log_{32}{a})}=4$ then a must lie in the range

a) $2<a<3$

b) $3<a<4$

c) $4<a<5$

d) $a>5$

3) Answer (C)

View Video Solution

Solution:

We have :$\frac{\log_{15}{a}+\log_{32}{a}}{(\log_{15}{a})(\log_{32}{a})}=4$
We get $\frac{\left(\frac{\log a}{\log\ 15}+\frac{\log a}{\log32}\right)}{\frac{\log a}{\log\ 15}\times\ \frac{\log a}{\log32}\ \ }=4$
we get $\log a\left(\log32\ +\log\ 15\right)=4\left(\log\ a\right)^2$
we get $\left(\log32\ +\log\ 15\right)=4\log a$
=$\log480=\log a^4$
=$a^4\ =480$
so we can say a is between 4 and 5 .

Question 4: If $\log_{2}[3+\log_{3} \left\{4+\log_{4}(x-1) \right\}]-2=0$ then 4x equals

4) Answer: 5

View Video Solution

Solution:

We have :
$\log_2\left\{3+\log_3\left\{4+\log_4\left(x-1\right)\right\}\right\}=2$
we get $3+\log_3\left\{4+\log_4\left(x-1\right)\right\}=4$
we get $\log_3\left(4+\log_4\left(x-1\right)\ =\ 1\right)$
we get $4+\log_4\left(x-1\right)\ =\ 3$
$\log_4\left(x-1\right)\ =\ -1$
x-1 = 4^-1
x = $\frac{1}{4}+1=\frac{5}{4}$
4x = 5

Question 5: If $5 – \log_{10}\sqrt{1 + x} + 4 \log_{10} \sqrt{1 – x} = \log_{10} \frac{1}{\sqrt{1 – x^2}}$, then 100x equals

5) Answer: 99

View Video Solution

Solution:

$5 – \log_{10}\sqrt{1 + x} + 4 \log_{10} \sqrt{1 – x} = \log_{10} \frac{1}{\sqrt{1 – x^2}}$

We can re-write the equation as: $5-\log_{10}\sqrt{1+x}+4\log_{10}\sqrt{1-x}=\log_{10}\left(\sqrt{1+x}\times\ \sqrt{1-x}\right)^{-1}$

$5-\log_{10}\sqrt{1+x}+4\log_{10}\sqrt{1-x}=\left(-1\right)\log_{10}\left(\sqrt{1+x}\right)+\left(-1\right)\log_{10}\left(\sqrt{1-x}\right)$

$5=-\log_{10}\sqrt{1+x}+\log_{10}\sqrt{1+x}-\log_{10}\sqrt{1-x}-4\log_{10}\sqrt{1-x}$

$5=-5\log_{10}\sqrt{1-x}$

$\sqrt{1-x}=\frac{1}{10}$

Squaring both sides: $\left(\sqrt{1-x}\right)^2=\frac{1}{100}$

$\therefore\ $ $x=1-\frac{1}{100}=\frac{99}{100}$

Hence, $100\ x\ =100\times\ \frac{99}{100}=99$

Enroll to SNAP & NMAT 2022 Crash Course

Question 6: If $\log \left(\frac{a}{b}\right) + \log \left(\frac{b}{a}\right) = \log(a + b)$, then which of the following statements is CORRECT?

a) a – b = 1

b) a + b = 1

c) a = b

d) $a^2 – b^2 = 1$

6) Answer (B)

Solution:

log(a/b) + log(b/a) = log(a+b)

log(a+b) = log(a/b)(b/a)

log(a+b)=log 1

a+b=1

Question 7: If $\log_4m + \log_4n = \log_2(m + n)$ where m and n are positive real numbers, then which of the following must be true?

a) $\frac{1}{m} + \frac{1}{n} = 1$

b) m = n

c) $m^2 + n^2 = 1$

d) $\frac{1}{m} + \frac{1}{n} = 2$

e) No values of m and n can satisfy the given equation

7) Answer (E)

Solution:

$\log_4mn=\log_2(m+n)$

$\sqrt{\ mn}=(m+n)$

Squarring on both sides

$m^2+n^2+mn\ =\ 0$

Since m, n are positive real numbers, no value of m and n satisfy the above equations.

Question 8: The value of $\log_{a}({\frac{a}{b}})+\log_{b}({\frac{b}{a}})$, for $1<a\leq b$ cannot be equal to

a) 0

b) -1

c) 1

d) -0.5

8) Answer (C)

View Video Solution

Solution:

On expanding the expression we get $1-\log_ab+1-\log_ba$

$or\ 2-\left(\log_ab+\frac{1}{\log_ba}\right)$

Now applying the property of AM>=GM, we get that  $\frac{\left(\log_ab+\frac{1}{\log_ba}\right)}{2}\ge1\ or\ \left(\log_ab+\frac{1}{\log_ba}\right)\ge2$ Hence from here we can conclude that the expression will always be equal to 0 or less than 0. Hence any positive value is not possible. So 1 is not possible.

Question 9: $\frac{2\times4\times8\times16}{(\log_{2}{4})^{2}(\log_{4}{8})^{3}(\log_{8}{16})^{4}}$ equals

9) Answer: 24

View Video Solution

Solution:

$\frac{\left(2\cdot4\cdot8\cdot16\right)}{\left(\log_22^2\right)^2\cdot\left(\log_{2^2}2^3\right)^3\cdot\left(\log_{2^3}2^4\right)^4}\cdot$

= $\frac{2^{10}}{4\cdot\left(\frac{3}{2}\right)^3\cdot\left(\frac{4}{3}\right)^4}=24$

Question 10: If $\log_{a}{30}=A,\log_{a}({\frac{5}{3}})=-B$ and $\log_2{a}=\frac{1}{3}$, then $\log_3{a}$ equals

a) $\frac{2}{A+B-3}$

b) $\frac{2}{A+B}-3$

c) $\frac{A+B}{2}-3$

d) $\frac{A+B-3}{2}$

10) Answer (A)

View Video Solution

Solution:

$\log_a30=A\ or\ \log_a5+\log_a2+\log_a3=A$………..(1)

$\log_a\left(\frac{5}{3}\right)=-B\ or\ \log_a3-\log_a5=B$………….(2)

and finally $\log_a2=3$

Substituting this in (1) we get $\log_a5+\log_a3=A-3$

Now we have two equations in two variables (1) and (2) . On solving we get

$\log_a3=\frac{\left(A+B-3\right)}{2\ }or\ \log_3a=\frac{2}{A+B-3}$

Take  SNAP mock tests here

Enrol to 10 SNAP Latest Mocks For Just Rs. 499

Question 11: If $\log_{4}{5}=(\log_{4}{y})(\log_{6}{\sqrt{5}})$, then y equals

11) Answer: 36

View Video Solution

Solution:

$\frac{\log\ 5}{2\log2}\ =\frac{\log\ y}{2\log2}\cdot\frac{\log\ 5}{2\log6}$

$\log\ 36\ =\ \log\ y;\ \therefore\ y\ =36$

Question 12: If Y is a negative number such that $2^{Y^2({\log_{3}{5})}}=5^{\log_{2}{3}}$, then Y equals to:

a) $\log_{2}(\frac{1}{5})$

b) $\log_{2}(\frac{1}{3})$

c) $-\log_{2}(\frac{1}{5})$

d) $-\log_{2}(\frac{1}{3})$

12) Answer (B)

View Video Solution

Solution:

$2^{Y^2({\log_{3}{5})}}=5^{Y^2(\log_3 2)}$

Given, $5^{Y^2\left(\log_32\right)}=5^{\left(\log_23\right)}$

=> $Y^2\left(\log_32\right)=\left(\log_23\right)=>Y^2=\left(\log_23\right)^2$

=>$Y=\left(-\log_23\right)^{\ }or\ \left(\log_23\right)$

since Y is a negative number, Y=$\left(-\log_23\right)=\left(\log_2\frac{1}{3}\right)$

Question 13: Let x and y be positive real numbers such that
$\log_{5}{(x + y)} + \log_{5}{(x – y)} = 3,$ and $\log_{2}{y} – \log_{2}{x} = 1 – \log_{2}{3}$. Then $xy$ equals

a) 150

b) 25

c) 100

d) 250

13) Answer (A)

View Video Solution

Solution:

We have, $\log_{5}{(x + y)} + \log_{5}{(x – y)} = 3$

=> $x^2-y^2=125$……(1)

$\log_{2}{y} – \log_{2}{x} = 1 – \log_{2}{3}$

=>$\ \frac{\ y}{x}$ = $\ \frac{\ 2}{3}$

=> 2x=3y   => x=$\ \frac{\ 3y}{2}$

On substituting the value of x in 1, we get

$\ \frac{\ 5x^2}{4}$=125

=>y=10, x=15

Hence xy=150

Question 14: Sham is trying to solve the expression:
$\log \tan 1^\circ + \log \tan 2^\circ + \log \tan 3^\circ + …….. + \log \tan 89^\circ$.
The correct answer would be?

a) 1

b) $\frac{1}{\sqrt{2}}$

c) 0

d) -1

14) Answer (C)

Solution:

$\log \tan 1^\circ + \log \tan 2^\circ + \log \tan 3^\circ + …….. + \log \tan 89^\circ$.

=$\log \tan 1^\circ + \log \tan 89^\circ + \log \tan 2^\circ + \log \tan 88^\circ …….. + \log \tan 45^\circ$.

=$\log\ \left(\tan\ 1^0\cdot\tan\ 89^0\right)\times\log\ \left(\tan\ 2^0\cdot\tan\ 88^0\right)\ ………………………\log\ \left(\tan\ 45^0\right)$

tan $45^0$ = 1

$\log\ \left(\tan\ 45^0\right)\ =\ 0$

$\therefore$ $\log \tan 1^\circ + \log \tan 2^\circ + \log \tan 3^\circ + …….. + \log \tan 89^\circ$ = 0

Question 15: If $\log_{10}{11} = a$ then $\log_{10}{\left(\frac{1}{110}\right)}$ is equal to

a) $-a$

b) $(1 + a)^{-1}$

c) $\frac{1}{10 a}$

d) $-(a + 1)$

15) Answer (D)

Solution:

$\log_{10}{\left(\frac{1}{110}\right)}$

$\log_a\left(\ \frac{\ x}{y}\right)\ =\ \log_ax-\log_ay$

$\log_{10}{\left(\frac{1}{110}\right)}$ = $=\ \log_{10}1-\log_{10}110$

= 0$-\log_{10}110$

=$-\log_{10}11\times\ 10$

=$-\left(\log_{10}11+\log_{10}10\right)$

= -(a+1)

D is the correct answer.

Question 16: Find the value of $\log_{10}{10} + \log_{10}{10^2} + ….. + \log_{10}{10^n}$

a) $n^{2} + 1$

b) $n^{2} – 1$

c) $\frac{(n^{2} + n)}{2}.\frac{n(n + 1)}{3}$

d) $\frac{(n^{2} + n)}{2}$

16) Answer (D)

Solution:

$\log_{10}{10} + \log_{10}{10^2} + ….. + \log_{10}{10^n}$

Since $\log_aa\ $ = 1

$\log_{10}{10} + \log_{10}{10^2} + ….. + \log_{10}{10^n}$ = 1+2+….n

=$\ \frac{\ n\left(n+1\right)}{2}$

=$\frac{(n^{2} + n)}{2}$

D is the correct answer.

Question 17: what is the value of $\frac{\log_{27}{9} \times \log_{16}{64}}{\log_{4}{\sqrt2}}$?

a) $\frac{1}{6}$

b) $\frac{1}{4}$

c) 8

d) 4

17) Answer (D)

Solution:

$\frac{\log_{27}{9} \times \log_{16}{64}}{\log_{4}{\sqrt2}}$?

=$\frac{\ \log_{3^3}3^2\times\ \log_{2^4}2^6}{\log_{\left(\sqrt{\ 2}\right)^4}\sqrt{\ 2}}$

=$\frac{\ \ \frac{\ 2}{3}\times\ \frac{\ 6}{4}}{\ \frac{\ 1}{4}}$

=4

D is the correct answer.

Question 18: What is the value of x in the following expression?
$x + \log_{10} (1 + 2^x) = x \log_{10} 5 + \log_{10} 6$

a) 1

b) 0

c) -1

d) 3

18) Answer (A)

Solution:

The given equation can be written as

$\log\left(10\right)^{x\ }\ +\ \log\left(1+2^x\right)=\log\left(5\right)^x+\log6$

$\log\left(10\right)^{x\ }\left(1+2^x\right)=\log\left(5\right)^x\cdot6$    (  since logA + logB=logAB)

$\log\ \frac{\left(2^x\cdot5^x\right)\left(1+2^x\right)}{5^x\cdot6}=0$    ( since logA – logB=logA/B)

$\frac{\left(2^x\ +2^{2x\ }\right)}{6}=10^0$  ($Since\ \log_aN\ =x\ \ =>N=a^x$)

$2^{^x}+2^{2x}=6$

The above  equation is satisfied only when x=1

Question 19: Find the value of $\log_{3^2}{5^4} \times \log_{5^2}{3^4}$

a) 5

b) 3

c) 4

d) 2

19) Answer (C)

Solution:

$\log_{b^n}\left(a^m\right)\ =\frac{m}{n}\log_ba\ =\frac{m}{n}\cdot\frac{\log\left(a\right)}{\log\left(b\right)}$

So given equation becomes $\frac{4}{2}\cdot\frac{4}{2}\cdot\frac{\log\left(3\right)}{\log\left(2\right)}\cdot\frac{\log\left(2\right)}{\log\left(3\right)}$  = 4

Question 20: $\log_{5}{25} + \log_{2} (\log_{3}{81})$ is

a) 1

b) 2

c) 3

d) 4

20) Answer (D)

Solution:

$\log\left(a^m\right)\ =\ m\log\left(a\right)\ and\ \ \log_aa$ = 1

$\log_55^2\ +\ \log_2\left(\log_33^4\right)$

2 + $\ \log_24$

2+ $\ \log_22^2$

4

Enroll to SNAP & NMAT 2022 Crash Course

Enroll to CAT 2022 course

LEAVE A REPLY

Please enter your comment!
Please enter your name here